יום שישי, 30 במרץ 2007

שאלות מכאניות

שלוש שאלות בסיסיות במכניקה של היומיום: כוח דימיוני,תנועת אופניים ושקול ניוטוני
מאת: יונתן שנהב

יום שבת 31 מרץ 2007
הפעם 3 שאלות ברמה בסיסית יותר, אבל מעניינות לא פחות
האם קיים כוח צנטרפוגלי? אם לא, מהו אותו כוח דמיוני שאנו מרגישים?

אין כוח כזה . באופן כללי ניתן לומר שהכוח הצנטרפוגלי הוא תוצר לוואי של תאוצה ולא הגורם שלו. אולם כל אחד מרגיש את הכוח הזה בזמן פנייה של האוטו. זהו כוח האינרציה שגורם לנו לנוע בקו ישר אל מחוץ למעגל התנועה. הכוח הצנטרפיטלי מושך אותנו פנימה. ע“מ להמחיש את ההבדל בין כוח רגיל וכוח צנטרפוגלי נדמיין שאנחנו החפץ המסובב ע“י חבל. הכוחות במערכת הם האינרציה שדוחפת אותך החוצה מהמעגל בקו ישר, הכוח של החבל מושך אותך פנימה, וצירוף שניהם יוצר (ע“י חיבור ווקטורי של רכיבים מאונכים) מעגל. אולם אין שום כוח שמושך אותך החוצה מהמעגל, בכיוון מנוגד לכיוון משיכת החבל. למעשה ניתן להגיד שהחבל גורם לך להאיץ אבל בכיוון שונה מהכיוון שהוא מושך, וזאת כתוצאה מחיבור מהירויות. דוגמא נוספת לזו ניתן לראות אם נירה את עצמנו מתותח. אנו מאיצים לכיוון מרכז כדה“א אולם נעים באלכסון למטה ולא אנכית ישירות.
למה אופניים נופלים בעמידה ולא נופלים אם נסיע אותם במהירות?


כשאנו רוכבים על אופניים אנו יוצרים מהירות סיבובית בעלת וקטור הפונה (לפי חוק יד ימין) לצד שמאל שלנו. יצרנו מומנטום זוויתי, גורם קבוע ביקום, ועל בסיס שימורו נוכיח שהאופניים יסטו לכיוון המתאים במקרה של נפילה. נניח שהאופניים רוצים ליפול שמאלה. כעת הכוח שמפעילה הריצפה (על הגלגל הנטוי בזווית שמאלה) יוצר מומנט בעל ווקטור בכיוון אחורה של האופניים (שוב, בשימוש חוק יד ימין). הכוח הוא הנורמל, הזרוע היא המרחק בין הגלגל שנוגע בקרקע ומרכז המסה של האופניים. להמחשה, כל ילד יודע שלא ניתן להביא את מרכז הכובד של הגוף מעבר לקצה הציפורן , כי מעבר לנקודה זו ניפול. במקרה של האופניים השקול יהיה שמאלה ואחורה. ע“מ לקזז את הרכיב שנוסף (שהרי קבענו שהמומנטום חייב להישמר קבוע) האופניים מוסטים שמאלה. כעת שוב הרכיב היחיד של התנע הסיבובי הוא השמאלה החדש שלנו.
אם קטר מפעיל כוח שווה לזה שקרון מפעיל עליו מדוע הם זזים? החוק השלישי של ניוטון קובע שאם מופעל על גוף כוח, הגוף מתנגד בכוח שווה בגודלו והפוך בכיוונו. אם שניהם מפעילים כוחות שווים האם הם לא אמורים לעמוד במקום? השאלה הבסיסית שאמורה להישאל היא מהי מערכת הצירים בה אנו משתמשים. אם נציב את מערכת הצירים על הרכבת אז ישנם שני כוחות: כח המנוע וכוח משיכת הקרון. עבור מערכת המוצבת על הקרון יש רק את כח הרכבת המושכת. ההבדל הוא בכוחות החיצוניים בכל המערכות. יש לראות את המנוע ככוח חיצוני המופעל על הרכבת (נניח שהיא בעלת מסה זהה לקרון M ) ולא ככזה שמושך את הקרון. כוח המנוע (F) מזיז 2M בתאוצה קבועה a . מכיוון שהקרון נע בתאוצה a, הכוח שמושכת אותו הרכבת הוא Ma . אז נכון שהכוחות שהרכבת והקרון מקזזים אחד את השני אולם על הרכבת מופעל כוח גדול פי שניים. אם נעבור לחיים שלנו , במקרה של אנשים שדוחפים אחד השני בתור לבנק, ניתן להתייחס להבדלים בין החיכוך שהרגליים שלהם מפעילות על הריצפה. ואם החיכוך שהשני מפעיל על הריצפה זהה בגודלו לחיכוך שהראשון מפעיל על הריצפה - אז כן, הם לא יזוזו עד מחר. כל ההבדל הוא בכוחות החיצוניים - מנוע, חיכוך עם הריצפה, יד שמחזיקה משהו. למשל בהורדות ידיים המנצח הוא זה שהיד שלא מתחרה במשיכת ידיים עצמה מושכת הכי חזק את הידית בצד השולחן. נשמע מוזר? נוכל לראות את שני המתחרים כמערכת אחת. למערכת זו יש שני כוחות חיצוניים בלבד: 2 הידיים שמחזיקות ידיות (נזניח חיכוך עם הריצפה, או יותר נכון נכניס את תרומתו לכוח של היד השניה) . אם כוחות אלו שווים (והידיים של המתחרים שוות באורכן) המתחרים ישארו בשו“מ סטטי. אם כוח א‘ גדול מכוח ב‘ אזי כוח ב‘ לא יוכל למנוע מהמערכת לזוז, שקול המומנטים יהיה שונה מאפס, וכל המערכת תתפתל בכיוון שכוח א‘ מניע אליו. נוכל כמובן להתמודד בלי ידיים, ואז ינצח מי שמפעיל כוח גדול יותר עם הנעליים שלו על הריצפה. השורה התחתונה היא פשוטה: כשאתה מפעיל פיתול על זרוע היריב גופך חייב להתפתל בכיוון השני, באופן שיקזז את הופעת התנע הזוויתי. נוכל לראות תופעה זו בנדנדה. כשילד מנדנד עצמו קדימה נוצר וקטור תנע זוויתי הפונה ימינה (לפי חוק יד ימין). אבל שרשרת אינה כמו נעל הנוגעת בריצפה, כלומר יציבה ומסוגלת להפעיל מומנט או כוח. שרשרת יכולה להפעיל רק כח אנכי. לכן הילד ש“יזרוק“ את רגליו קדימה תמיד ירגיש את ראשו נע אחורה, ובכך נשמר התנע הזוויתי ביקום.

אין תגובות: